Strengthen the argument

This topic has expert replies
Master | Next Rank: 500 Posts
Posts: 106
Joined: Sat Mar 30, 2013 11:55 pm
Thanked: 1 times
Followed by:1 members

Strengthen the argument

by veenu08 » Sun Jun 09, 2013 6:00 am
When the nineteenth-century German bacteriologist Robert Koch identified a particular bacterium as responsible for cholera, Max von Pettenkoffer, a physician, expressed his skepticism by voluntarily drinking an entire bottle of the allegedly responsible bacteria. Although von Pettenkoffer took his failure to come down with the disease as a refutation of Koch's hypothesis that cholera was caused by bacteria, Koch argued that von Pettenkoffer had been protected by his own stomach acid. The acid secreted by the stomach, Koch explained, kills most ingested bacteria.

Which of the following, if true, provides the most evidence to support Koch's counterargument?

A.Peptic ulcers, often associated with excessive secretions of stomach acid, are common in certain areas characterized by low rates of cholera.
B.As von Pettenkoffer later admitted that he had previously had cholera, it is probable that he had developed antibodies that protected him from a second attack.
C. Cholera is endemic in areas in which poor sanitation results in high concentrations of cholera bacteria in drinking water.
D. Although stomach acid kills most ingested bacteria, large numbers of e. coli bacteria nonetheless manage to make their way to the lower intestine of the digestive tract.
E.Cholera bacteria ingested with bicarbonate of soda, a neutralizer of stomach acid, is more likely to result in cholera than if the bacteria is ingested alone.

Source Kaplan Diagnostic Test

OA E

User avatar
Senior | Next Rank: 100 Posts
Posts: 48
Joined: Sun Jan 15, 2012 11:34 am

by theunheardmelody » Mon Jun 17, 2013 9:21 am
Good one.

You can eliminate A, C and D by POE since they dont directly pertain to the arguement.

Between B and E

B - Althought it seems right and make sense the question is to find a statement that supports Koch's arguement (related to stomach acids) Only E does that.

Hence E is the answer choice.

Hope this helps.

can I also request that you dont show the OA right away. Maybe add/share once you have a few responses :) ..

Cheers